LSAT 16 – Section 3 – Question 06

You need a full course to see this video. Enroll now and get started in less than a minute.

Target time: 1:08

This is question data from the 7Sage LSAT Scorer. You can score your LSATs, track your results, and analyze your performance with pretty charts and vital statistics - all with a Free Account ← sign up in less than 10 seconds

Question
QuickView
Type Tags Answer
Choices
Curve Question
Difficulty
Psg/Game/S
Difficulty
Explanation
PT16 S3 Q06
+LR
Weaken +Weak
A
1%
157
B
9%
159
C
80%
168
D
1%
167
E
10%
164
130
146
161
+Medium 147.952 +SubsectionMedium
This page shows a recording of a live class. We're working hard to create our standard, concise explanation videos for the questions in this PrepTest. Thank you for your patience!

This is a weakening question, we know because the question stem says: Which one of the following, if true, most undermines the author’s prediction?

This is a nice and short stimulus; some unexpected heavy rainfall has filled reservoirs and streams, and therefore the author predicts that it won’t be necessary to ration water this summer. The assumption underlying the argument is a conditional; if the reservoirs and streams are full, then water rationing won’t be necessary. The correct answer is going to undermine this hidden premise that the reservoirs and streams being full is enough for water rationing not to be needed. Like many weakening questions, there is a lot of ways to come at this argument, and therefore POE is the best way to approach the answer choices. Let’s take a look at them:

Answer Choice (A)This is a poor answer because it relies on inductive reasoning (what happened in the past will happen in the future), and the author specifies that the reservoirs and streams being filled by heavy rainfall is a recent and unexpected phenomenon. What happened in previous years is not relevant to the support for the prediction.

Answer Choice (B) This is a very weak answer. For one, it admits that only a small part of the city’s water supply comes from this underground supply; you would expect that it would have little effect on whether water rationing will be required. All this answer does is introduce another supply in addition to the full reservoirs we’ve been told about, which would strengthen the prediction that water rationing won’t be necessary.

Correct Answer Choice (C)This answer does exactly what we identified in the stimulus; it undermines the sufficiency of full reservoirs for a lack of water rationing. If the actual transportation of the water to individuals is the issue, then a surplus of water isn’t enough to prevent water shortages among customers, and therefore it is entirely possible water rationing will be necessary even if the water supply is full.

Answer Choice (D) This answer doesn’t help us because we don’t know much about the relation of temperature to water usage. If this instead said “long-range weather forecasts predicts an extremely hot summer, which may increase water usage” then it might be helpful. But as it is stated, D fails to weaken the prediction.

Answer Choice (E) This answer is similar to A in that it relies on the inductive assumption that what happens in most years will happen in future ones. Even if this answer guaranteed that there would be less rain in the summer, for all we know the full reservoirs could supply the city for another year without any rain.

Take PrepTest

Review Results

Leave a Reply